having trouble reconciling answer explanation w...
Answer explanation states "Here the philosopher is not saying that Graham is wrong" but t...
Nativeguy on June 20 at 11:52PM
  • September 2007 LSAT
  • SEC3
  • Q17
1
Reply
Could you please explain why D and E are wrong.
Thanks, I see someone also asked about D and has yet to get a response.
DylanMorris on September 16, 2022
  • September 2007 LSAT
  • SEC3
  • Q23
1
Reply
How is C correct compared to D?
C states, "fails to take into account the possibility that patients at Edgewater Hospital tend to...
Tyler808 on August 5, 2022
  • September 2007 LSAT
  • SEC3
  • Q16
3
Replies
Answer C
Why is C the right answer? Thank you
filozinni on March 23, 2022
  • September 2007 LSAT
  • SEC3
  • Q6
5
Replies
Clarification
I can't understand from the passage how the first sentence is true. Could you please break down t...
tselimovic on February 19, 2021
  • September 2007 LSAT
  • SEC3
  • Q23
4
Replies
Answer D
Why is answer D correct? Thank you!
filozinni on October 29, 2020
  • September 2007 LSAT
  • SEC3
  • Q15
3
Replies
Explanation
Can someone please explain the right answer?
zia305 on August 20, 2020
  • September 2007 LSAT
  • SEC3
  • Q2
3
Replies
Why not A?
Why not A? The correct answer is E and it is one I eliminated early on.
EugeneC on August 1, 2020
  • September 2007 LSAT
  • SEC3
  • Q10
2
Replies
Necessary Assumption
Please could you explain this argument and conclusion? This is a strengthen with necessary premis...
Anna20 on July 23, 2020
  • September 2007 LSAT
  • SEC3
  • Q13
1
Reply
Cause and Error mistake
Hi, I chose answer choice C. The other choices didn't make sense to me. During the office ho...
Jade on July 6, 2020
  • September 2007 LSAT
  • SEC3
  • Q4
1
Reply
Conclusion of Argument
Please could you clarify/explain why the answer for this question is B, rather than D? Many thanks!
Anna20 on June 13, 2020
  • September 2007 LSAT
  • SEC3
  • Q17
2
Replies
Answer choice C
Why is C correct?
filozinni on June 3, 2020
  • September 2007 LSAT
  • SEC3
  • Q9
2
Replies
Why A, not B or C.
My first choice was B, then C. I don't understand why is it A? Please help.
Alyona1983 on January 25, 2020
  • September 2007 LSAT
  • SEC3
  • Q25
2
Replies
Why not B
I can understand why E is the correct answer, but B looks as good to me. Please explain why B is ...
Alyona1983 on January 24, 2020
  • September 2007 LSAT
  • SEC3
  • Q22
2
Replies
Question
Hey! I put A as my answer. Why is D correct? Thanks!
claire_crites on April 30, 2019
  • September 2007 LSAT
  • SEC3
  • Q18
1
Reply
Help
Can you explain why A is the correct answer?
JayDee8732 on August 28, 2017
  • September 2007 LSAT
  • SEC3
  • Q7
1
Reply
Help
I don't understand why A is correct
JayDee8732 on August 28, 2017
  • September 2007 LSAT
  • SEC3
  • Q8
1
Reply
Can you please clarify?
The correct answer choice is C, which states that the reasoning fails to consider that patients t...
NicoCapri on September 26, 2014
  • September 2007 LSAT
  • SEC3
  • Q16
1
Reply
Parachuting
Of-> R ?x -M- 1ef >= 10x -S- f ????????
Lily on August 13, 2013
  • September 2007 LSAT
  • SEC3
  • Q21
1
Reply
Demand of domestic vs imported wine
I get this LR question, why the wrong answer choices are wrong and its answer is right ... Howev...
Lily on August 13, 2013
  • September 2007 LSAT
  • SEC3
  • Q22
1
Reply